2021 AMC 10A Problems/Problem 3

Revision as of 15:02, 6 January 2021 by Cjkmss (talk | contribs) (Solution)

Problem

These problems will not be released until the 2021 AMC 10A is released.

Solution

We don't know yet, but since 10+9=21, we know that the answer is G.

Note

This problem might also be on the AMC 12A. If so, please redirect it there.

See also

2021 AMC 10A (ProblemsAnswer KeyResources)
Preceded by
Problem 2
Followed by
Problem 4
1 2 3 4 5 6 7 8 9 10 11 12 13 14 15 16 17 18 19 20 21 22 23 24 25
All AMC 10 Problems and Solutions

The problems on this page are copyrighted by the Mathematical Association of America's American Mathematics Competitions. AMC logo.png